How strong is the axiom of well-ordered choice?No uncountable ordinals without the axiom of choice?Axiom of...

Help! My Character is too much for her story!

Why does a car's steering wheel get lighter with increasing speed

Is it a Cyclops number? "Nobody" knows!

I am the light that shines in the dark

Does an unused member variable take up memory?

Was this cameo in Captain Marvel computer generated?

Short story about an infectious indestructible metal bar?

Trigger on Custom Object Share

School performs periodic password audits. Is my password compromised?

Create chunks from an array

What does "rhumatis" mean?

Why is there an extra space when I type "ls" on the Desktop?

Unfamiliar notation in Diabelli's "Duet in D" for piano

Can Witch Sight see through Mirror Image?

“I had a flat in the centre of town, but I didn’t like living there, so …”

Generating a list with duplicate entries

What exactly is the meaning of "fine wine"?

Does the US political system, in principle, allow for a no-party system?

Unidentified signals on FT8 frequencies

An Undercover Army

What is better: yes / no radio, or simple checkbox?

How to educate team mate to take screenshots for bugs with out unwanted stuff

Do I need a return ticket to Canada if I'm a Japanese National?

What is the purpose of a disclaimer like "this is not legal advice"?



How strong is the axiom of well-ordered choice?


No uncountable ordinals without the axiom of choice?Axiom of choice , Hartogs ordinals, well-ordering principleWhich sets are well-orderable without Axiom of Choice?Well-orderings of $mathbb R$ without Choice“There is no well-ordered uncountable set of real numbers”Axiom of choice and the well ordering principleThe class of well-founded sets satisfies the axiom of foundation and the axiom of choiceAxiom of Choice is equivalent to Well-ordering Theorem: Hrbacek, Jech - “Introduction to Set Theory”the power set of every well-ordered set is well-ordered implies well orderingHow strong are weak choice principles?













8












$begingroup$


I sometimes see references to the "Axiom of Well-Ordered Choice," but I'm not sure how strong it is. It states that every well-ordered family of sets has a choice function.



By "well-ordered family," I don't mean that the sets within the family are well-ordered, but that the family must index all the sets within the family by some ordinal.



How strong is this axiom? Can it prove the Hahn-Banach theorem, the ultrafilter lemma, anything about measurable sets, etc? Does it have any implications about what sets can be well-ordered (the reals for instance), or perhaps prove anything about the Hartogs number of sets, etc?



Does anyone have a reference for this?










share|cite|improve this question











$endgroup$












  • $begingroup$
    ...isn't this axiom a consequence of ZF? One can choose the (well-defined, since it's well-ordered) lexicographically least element of each set in the family...
    $endgroup$
    – Steven Stadnicki
    4 hours ago






  • 2




    $begingroup$
    That is a family of well-ordered sets, not a well-ordered family of (arbitrary sets).
    $endgroup$
    – Mike Battaglia
    4 hours ago










  • $begingroup$
    Ahh, I missed that distinction. Thank you!
    $endgroup$
    – Steven Stadnicki
    4 hours ago










  • $begingroup$
    I've never seen this axiom before. Does the family itself need to be a set or can it be a proper class?
    $endgroup$
    – Robert Shore
    4 hours ago










  • $begingroup$
    Google suggests this: settheory.mathtalks.org/andreas-blass-well-ordered-choice
    $endgroup$
    – Carl Mummert
    4 hours ago
















8












$begingroup$


I sometimes see references to the "Axiom of Well-Ordered Choice," but I'm not sure how strong it is. It states that every well-ordered family of sets has a choice function.



By "well-ordered family," I don't mean that the sets within the family are well-ordered, but that the family must index all the sets within the family by some ordinal.



How strong is this axiom? Can it prove the Hahn-Banach theorem, the ultrafilter lemma, anything about measurable sets, etc? Does it have any implications about what sets can be well-ordered (the reals for instance), or perhaps prove anything about the Hartogs number of sets, etc?



Does anyone have a reference for this?










share|cite|improve this question











$endgroup$












  • $begingroup$
    ...isn't this axiom a consequence of ZF? One can choose the (well-defined, since it's well-ordered) lexicographically least element of each set in the family...
    $endgroup$
    – Steven Stadnicki
    4 hours ago






  • 2




    $begingroup$
    That is a family of well-ordered sets, not a well-ordered family of (arbitrary sets).
    $endgroup$
    – Mike Battaglia
    4 hours ago










  • $begingroup$
    Ahh, I missed that distinction. Thank you!
    $endgroup$
    – Steven Stadnicki
    4 hours ago










  • $begingroup$
    I've never seen this axiom before. Does the family itself need to be a set or can it be a proper class?
    $endgroup$
    – Robert Shore
    4 hours ago










  • $begingroup$
    Google suggests this: settheory.mathtalks.org/andreas-blass-well-ordered-choice
    $endgroup$
    – Carl Mummert
    4 hours ago














8












8








8


1



$begingroup$


I sometimes see references to the "Axiom of Well-Ordered Choice," but I'm not sure how strong it is. It states that every well-ordered family of sets has a choice function.



By "well-ordered family," I don't mean that the sets within the family are well-ordered, but that the family must index all the sets within the family by some ordinal.



How strong is this axiom? Can it prove the Hahn-Banach theorem, the ultrafilter lemma, anything about measurable sets, etc? Does it have any implications about what sets can be well-ordered (the reals for instance), or perhaps prove anything about the Hartogs number of sets, etc?



Does anyone have a reference for this?










share|cite|improve this question











$endgroup$




I sometimes see references to the "Axiom of Well-Ordered Choice," but I'm not sure how strong it is. It states that every well-ordered family of sets has a choice function.



By "well-ordered family," I don't mean that the sets within the family are well-ordered, but that the family must index all the sets within the family by some ordinal.



How strong is this axiom? Can it prove the Hahn-Banach theorem, the ultrafilter lemma, anything about measurable sets, etc? Does it have any implications about what sets can be well-ordered (the reals for instance), or perhaps prove anything about the Hartogs number of sets, etc?



Does anyone have a reference for this?







set-theory axiom-of-choice foundations well-orders






share|cite|improve this question















share|cite|improve this question













share|cite|improve this question




share|cite|improve this question








edited 3 hours ago







Mike Battaglia

















asked 4 hours ago









Mike BattagliaMike Battaglia

1,4271126




1,4271126












  • $begingroup$
    ...isn't this axiom a consequence of ZF? One can choose the (well-defined, since it's well-ordered) lexicographically least element of each set in the family...
    $endgroup$
    – Steven Stadnicki
    4 hours ago






  • 2




    $begingroup$
    That is a family of well-ordered sets, not a well-ordered family of (arbitrary sets).
    $endgroup$
    – Mike Battaglia
    4 hours ago










  • $begingroup$
    Ahh, I missed that distinction. Thank you!
    $endgroup$
    – Steven Stadnicki
    4 hours ago










  • $begingroup$
    I've never seen this axiom before. Does the family itself need to be a set or can it be a proper class?
    $endgroup$
    – Robert Shore
    4 hours ago










  • $begingroup$
    Google suggests this: settheory.mathtalks.org/andreas-blass-well-ordered-choice
    $endgroup$
    – Carl Mummert
    4 hours ago


















  • $begingroup$
    ...isn't this axiom a consequence of ZF? One can choose the (well-defined, since it's well-ordered) lexicographically least element of each set in the family...
    $endgroup$
    – Steven Stadnicki
    4 hours ago






  • 2




    $begingroup$
    That is a family of well-ordered sets, not a well-ordered family of (arbitrary sets).
    $endgroup$
    – Mike Battaglia
    4 hours ago










  • $begingroup$
    Ahh, I missed that distinction. Thank you!
    $endgroup$
    – Steven Stadnicki
    4 hours ago










  • $begingroup$
    I've never seen this axiom before. Does the family itself need to be a set or can it be a proper class?
    $endgroup$
    – Robert Shore
    4 hours ago










  • $begingroup$
    Google suggests this: settheory.mathtalks.org/andreas-blass-well-ordered-choice
    $endgroup$
    – Carl Mummert
    4 hours ago
















$begingroup$
...isn't this axiom a consequence of ZF? One can choose the (well-defined, since it's well-ordered) lexicographically least element of each set in the family...
$endgroup$
– Steven Stadnicki
4 hours ago




$begingroup$
...isn't this axiom a consequence of ZF? One can choose the (well-defined, since it's well-ordered) lexicographically least element of each set in the family...
$endgroup$
– Steven Stadnicki
4 hours ago




2




2




$begingroup$
That is a family of well-ordered sets, not a well-ordered family of (arbitrary sets).
$endgroup$
– Mike Battaglia
4 hours ago




$begingroup$
That is a family of well-ordered sets, not a well-ordered family of (arbitrary sets).
$endgroup$
– Mike Battaglia
4 hours ago












$begingroup$
Ahh, I missed that distinction. Thank you!
$endgroup$
– Steven Stadnicki
4 hours ago




$begingroup$
Ahh, I missed that distinction. Thank you!
$endgroup$
– Steven Stadnicki
4 hours ago












$begingroup$
I've never seen this axiom before. Does the family itself need to be a set or can it be a proper class?
$endgroup$
– Robert Shore
4 hours ago




$begingroup$
I've never seen this axiom before. Does the family itself need to be a set or can it be a proper class?
$endgroup$
– Robert Shore
4 hours ago












$begingroup$
Google suggests this: settheory.mathtalks.org/andreas-blass-well-ordered-choice
$endgroup$
– Carl Mummert
4 hours ago




$begingroup$
Google suggests this: settheory.mathtalks.org/andreas-blass-well-ordered-choice
$endgroup$
– Carl Mummert
4 hours ago










1 Answer
1






active

oldest

votes


















4












$begingroup$

The axiom of well-ordered choice, or $sf AC_{rm WO}$, is strictly weaker than the axiom of choice itself. If we start with $L$ and add $omega_1$ Cohen reals, then go to $L(Bbb R)$, one can show that $sf AC_{rm WO}$ holds, while $Bbb R$ cannot be well-ordered there.



Pincus proved in the 1970s that this is equivalent to the following statement on Hartogs and Lindenbaum numbers:




$sf AC_{rm WO}$ is equivalent to the statement $forall x.aleph(x)=aleph^*(x)$.




Here, the Lindenbaum number, $aleph^*(x)$, is the least ordinal which $x$ cannot be mapped onto. One obvious fact is that $aleph(x)leqaleph^*(x)$.



In the late 1950s or early 1960s Jensen proved that this assumption also implies $sf DC$. This is also a very clever proof.



The conjunction of these two consequences gives us that $aleph_1leq 2^{aleph_0}$, as a result of a theorem of Shelah from the 1980s, this implies there is a non-measurable set of reals.



As far as Hahn–Banach, or other things of that sort, I do not believe that much is known on the topic. But to sum up, this axiom does not imply that the reals are well-ordered, but it does imply there is a non-measurable set of reals because there is a set of reals of size $aleph_1$ and $sf DC$ holds. Moreover, it is equivalent to saying that the Hartogs and Lindenbaum numbers are equal for all sets.






share|cite|improve this answer









$endgroup$













  • $begingroup$
    Do you have a source for a proof of ${sf AC_{rm WO}}iff forall x(aleph(x)=aleph^*(x))$?
    $endgroup$
    – Holo
    2 hours ago












  • $begingroup$
    @Holo karagila.org/2014/on-the-partition-principle
    $endgroup$
    – Asaf Karagila
    1 hour ago













Your Answer





StackExchange.ifUsing("editor", function () {
return StackExchange.using("mathjaxEditing", function () {
StackExchange.MarkdownEditor.creationCallbacks.add(function (editor, postfix) {
StackExchange.mathjaxEditing.prepareWmdForMathJax(editor, postfix, [["$", "$"], ["\\(","\\)"]]);
});
});
}, "mathjax-editing");

StackExchange.ready(function() {
var channelOptions = {
tags: "".split(" "),
id: "69"
};
initTagRenderer("".split(" "), "".split(" "), channelOptions);

StackExchange.using("externalEditor", function() {
// Have to fire editor after snippets, if snippets enabled
if (StackExchange.settings.snippets.snippetsEnabled) {
StackExchange.using("snippets", function() {
createEditor();
});
}
else {
createEditor();
}
});

function createEditor() {
StackExchange.prepareEditor({
heartbeatType: 'answer',
autoActivateHeartbeat: false,
convertImagesToLinks: true,
noModals: true,
showLowRepImageUploadWarning: true,
reputationToPostImages: 10,
bindNavPrevention: true,
postfix: "",
imageUploader: {
brandingHtml: "Powered by u003ca class="icon-imgur-white" href="https://imgur.com/"u003eu003c/au003e",
contentPolicyHtml: "User contributions licensed under u003ca href="https://creativecommons.org/licenses/by-sa/3.0/"u003ecc by-sa 3.0 with attribution requiredu003c/au003e u003ca href="https://stackoverflow.com/legal/content-policy"u003e(content policy)u003c/au003e",
allowUrls: true
},
noCode: true, onDemand: true,
discardSelector: ".discard-answer"
,immediatelyShowMarkdownHelp:true
});


}
});














draft saved

draft discarded


















StackExchange.ready(
function () {
StackExchange.openid.initPostLogin('.new-post-login', 'https%3a%2f%2fmath.stackexchange.com%2fquestions%2f3140508%2fhow-strong-is-the-axiom-of-well-ordered-choice%23new-answer', 'question_page');
}
);

Post as a guest















Required, but never shown

























1 Answer
1






active

oldest

votes








1 Answer
1






active

oldest

votes









active

oldest

votes






active

oldest

votes









4












$begingroup$

The axiom of well-ordered choice, or $sf AC_{rm WO}$, is strictly weaker than the axiom of choice itself. If we start with $L$ and add $omega_1$ Cohen reals, then go to $L(Bbb R)$, one can show that $sf AC_{rm WO}$ holds, while $Bbb R$ cannot be well-ordered there.



Pincus proved in the 1970s that this is equivalent to the following statement on Hartogs and Lindenbaum numbers:




$sf AC_{rm WO}$ is equivalent to the statement $forall x.aleph(x)=aleph^*(x)$.




Here, the Lindenbaum number, $aleph^*(x)$, is the least ordinal which $x$ cannot be mapped onto. One obvious fact is that $aleph(x)leqaleph^*(x)$.



In the late 1950s or early 1960s Jensen proved that this assumption also implies $sf DC$. This is also a very clever proof.



The conjunction of these two consequences gives us that $aleph_1leq 2^{aleph_0}$, as a result of a theorem of Shelah from the 1980s, this implies there is a non-measurable set of reals.



As far as Hahn–Banach, or other things of that sort, I do not believe that much is known on the topic. But to sum up, this axiom does not imply that the reals are well-ordered, but it does imply there is a non-measurable set of reals because there is a set of reals of size $aleph_1$ and $sf DC$ holds. Moreover, it is equivalent to saying that the Hartogs and Lindenbaum numbers are equal for all sets.






share|cite|improve this answer









$endgroup$













  • $begingroup$
    Do you have a source for a proof of ${sf AC_{rm WO}}iff forall x(aleph(x)=aleph^*(x))$?
    $endgroup$
    – Holo
    2 hours ago












  • $begingroup$
    @Holo karagila.org/2014/on-the-partition-principle
    $endgroup$
    – Asaf Karagila
    1 hour ago


















4












$begingroup$

The axiom of well-ordered choice, or $sf AC_{rm WO}$, is strictly weaker than the axiom of choice itself. If we start with $L$ and add $omega_1$ Cohen reals, then go to $L(Bbb R)$, one can show that $sf AC_{rm WO}$ holds, while $Bbb R$ cannot be well-ordered there.



Pincus proved in the 1970s that this is equivalent to the following statement on Hartogs and Lindenbaum numbers:




$sf AC_{rm WO}$ is equivalent to the statement $forall x.aleph(x)=aleph^*(x)$.




Here, the Lindenbaum number, $aleph^*(x)$, is the least ordinal which $x$ cannot be mapped onto. One obvious fact is that $aleph(x)leqaleph^*(x)$.



In the late 1950s or early 1960s Jensen proved that this assumption also implies $sf DC$. This is also a very clever proof.



The conjunction of these two consequences gives us that $aleph_1leq 2^{aleph_0}$, as a result of a theorem of Shelah from the 1980s, this implies there is a non-measurable set of reals.



As far as Hahn–Banach, or other things of that sort, I do not believe that much is known on the topic. But to sum up, this axiom does not imply that the reals are well-ordered, but it does imply there is a non-measurable set of reals because there is a set of reals of size $aleph_1$ and $sf DC$ holds. Moreover, it is equivalent to saying that the Hartogs and Lindenbaum numbers are equal for all sets.






share|cite|improve this answer









$endgroup$













  • $begingroup$
    Do you have a source for a proof of ${sf AC_{rm WO}}iff forall x(aleph(x)=aleph^*(x))$?
    $endgroup$
    – Holo
    2 hours ago












  • $begingroup$
    @Holo karagila.org/2014/on-the-partition-principle
    $endgroup$
    – Asaf Karagila
    1 hour ago
















4












4








4





$begingroup$

The axiom of well-ordered choice, or $sf AC_{rm WO}$, is strictly weaker than the axiom of choice itself. If we start with $L$ and add $omega_1$ Cohen reals, then go to $L(Bbb R)$, one can show that $sf AC_{rm WO}$ holds, while $Bbb R$ cannot be well-ordered there.



Pincus proved in the 1970s that this is equivalent to the following statement on Hartogs and Lindenbaum numbers:




$sf AC_{rm WO}$ is equivalent to the statement $forall x.aleph(x)=aleph^*(x)$.




Here, the Lindenbaum number, $aleph^*(x)$, is the least ordinal which $x$ cannot be mapped onto. One obvious fact is that $aleph(x)leqaleph^*(x)$.



In the late 1950s or early 1960s Jensen proved that this assumption also implies $sf DC$. This is also a very clever proof.



The conjunction of these two consequences gives us that $aleph_1leq 2^{aleph_0}$, as a result of a theorem of Shelah from the 1980s, this implies there is a non-measurable set of reals.



As far as Hahn–Banach, or other things of that sort, I do not believe that much is known on the topic. But to sum up, this axiom does not imply that the reals are well-ordered, but it does imply there is a non-measurable set of reals because there is a set of reals of size $aleph_1$ and $sf DC$ holds. Moreover, it is equivalent to saying that the Hartogs and Lindenbaum numbers are equal for all sets.






share|cite|improve this answer









$endgroup$



The axiom of well-ordered choice, or $sf AC_{rm WO}$, is strictly weaker than the axiom of choice itself. If we start with $L$ and add $omega_1$ Cohen reals, then go to $L(Bbb R)$, one can show that $sf AC_{rm WO}$ holds, while $Bbb R$ cannot be well-ordered there.



Pincus proved in the 1970s that this is equivalent to the following statement on Hartogs and Lindenbaum numbers:




$sf AC_{rm WO}$ is equivalent to the statement $forall x.aleph(x)=aleph^*(x)$.




Here, the Lindenbaum number, $aleph^*(x)$, is the least ordinal which $x$ cannot be mapped onto. One obvious fact is that $aleph(x)leqaleph^*(x)$.



In the late 1950s or early 1960s Jensen proved that this assumption also implies $sf DC$. This is also a very clever proof.



The conjunction of these two consequences gives us that $aleph_1leq 2^{aleph_0}$, as a result of a theorem of Shelah from the 1980s, this implies there is a non-measurable set of reals.



As far as Hahn–Banach, or other things of that sort, I do not believe that much is known on the topic. But to sum up, this axiom does not imply that the reals are well-ordered, but it does imply there is a non-measurable set of reals because there is a set of reals of size $aleph_1$ and $sf DC$ holds. Moreover, it is equivalent to saying that the Hartogs and Lindenbaum numbers are equal for all sets.







share|cite|improve this answer












share|cite|improve this answer



share|cite|improve this answer










answered 3 hours ago









Asaf KaragilaAsaf Karagila

306k33437768




306k33437768












  • $begingroup$
    Do you have a source for a proof of ${sf AC_{rm WO}}iff forall x(aleph(x)=aleph^*(x))$?
    $endgroup$
    – Holo
    2 hours ago












  • $begingroup$
    @Holo karagila.org/2014/on-the-partition-principle
    $endgroup$
    – Asaf Karagila
    1 hour ago




















  • $begingroup$
    Do you have a source for a proof of ${sf AC_{rm WO}}iff forall x(aleph(x)=aleph^*(x))$?
    $endgroup$
    – Holo
    2 hours ago












  • $begingroup$
    @Holo karagila.org/2014/on-the-partition-principle
    $endgroup$
    – Asaf Karagila
    1 hour ago


















$begingroup$
Do you have a source for a proof of ${sf AC_{rm WO}}iff forall x(aleph(x)=aleph^*(x))$?
$endgroup$
– Holo
2 hours ago






$begingroup$
Do you have a source for a proof of ${sf AC_{rm WO}}iff forall x(aleph(x)=aleph^*(x))$?
$endgroup$
– Holo
2 hours ago














$begingroup$
@Holo karagila.org/2014/on-the-partition-principle
$endgroup$
– Asaf Karagila
1 hour ago






$begingroup$
@Holo karagila.org/2014/on-the-partition-principle
$endgroup$
– Asaf Karagila
1 hour ago




















draft saved

draft discarded




















































Thanks for contributing an answer to Mathematics Stack Exchange!


  • Please be sure to answer the question. Provide details and share your research!

But avoid



  • Asking for help, clarification, or responding to other answers.

  • Making statements based on opinion; back them up with references or personal experience.


Use MathJax to format equations. MathJax reference.


To learn more, see our tips on writing great answers.




draft saved


draft discarded














StackExchange.ready(
function () {
StackExchange.openid.initPostLogin('.new-post-login', 'https%3a%2f%2fmath.stackexchange.com%2fquestions%2f3140508%2fhow-strong-is-the-axiom-of-well-ordered-choice%23new-answer', 'question_page');
}
);

Post as a guest















Required, but never shown





















































Required, but never shown














Required, but never shown












Required, but never shown







Required, but never shown

































Required, but never shown














Required, but never shown












Required, but never shown







Required, but never shown







Popular posts from this blog

Paper upload error, “Upload failed: The top margin is 0.715 in on page 3, which is below the required...

Emraan Hashmi Filmografia | Linki zewnętrzne | Menu nawigacyjneGulshan GroverGulshan...

How can I write this formula?newline and italics added with leqWhy does widehat behave differently if I...